Difference between revisions of "2019 AIME I Problems/Problem 5"
(→Solution) |
m (→Solution) |
||
Line 4: | Line 4: | ||
==Solution== | ==Solution== | ||
− | We label a move from <math>(a,b)</math> to <math>(a,b-1)</math> as down (<math>D</math>), from <math>(a,b)</math> to <math>(a-1,b)</math> as left (<math>L</math>), and from <math>(a,b)</math> to <math>(a-1,b-1)</math> as slant (<math>S</math>). To arrive at <math>(0,0)</math> without arriving at an axis first, the particle must first go to <math>(1,1)</math> then do a slant move. The particle can arrive | + | We label a move from <math>(a,b)</math> to <math>(a,b-1)</math> as down (<math>D</math>), from <math>(a,b)</math> to <math>(a-1,b)</math> as left (<math>L</math>), and from <math>(a,b)</math> to <math>(a-1,b-1)</math> as slant (<math>S</math>). To arrive at <math>(0,0)</math> without arriving at an axis first, the particle must first go to <math>(1,1)</math> then do a slant move. The particle can arrive at <math>(1,1)</math> through any permutation of the following 4 different cases: <math>SSS</math>, <math>SSDL</math>, <math>SDLDL</math>, and <math>DLDLDL</math>. |
There is only <math>1</math> permutation of <math>SSS</math>. Including the last move, there are <math>4</math> possible moves, making the probability of this move <math>\frac{1}{3^4}</math>. | There is only <math>1</math> permutation of <math>SSS</math>. Including the last move, there are <math>4</math> possible moves, making the probability of this move <math>\frac{1}{3^4}</math>. |
Revision as of 22:54, 14 March 2019
Problem 5
A moving particle starts at the point and moves until it hits one of the coordinate axes for the first time. When the particle is at the point , it moves at random to one of the points , , or , each with probability , independently of its previous moves. The probability that it will hit the coordinate axes at is , where and are positive integers. Find .
Solution
We label a move from to as down (), from to as left (), and from to as slant (). To arrive at without arriving at an axis first, the particle must first go to then do a slant move. The particle can arrive at through any permutation of the following 4 different cases: , , , and .
There is only permutation of . Including the last move, there are possible moves, making the probability of this move .
There are permutations of , as the ordering of the two slants do not matter. There are possible moves, making the probability of this move .
There are permutations of , as the ordering of the two downs and two lefts do not matter. There are possible moves, making the probability of this move .
There are permutations of , as the ordering of the three downs and three lefts do not matter. There are possible moves, making the probability of this move .
Adding these, we get the total probability as . Therefore, the answer is .
Solution by Zaxter22
See Also
2019 AIME I (Problems • Answer Key • Resources) | ||
Preceded by Problem 4 |
Followed by Problem 6 | |
1 • 2 • 3 • 4 • 5 • 6 • 7 • 8 • 9 • 10 • 11 • 12 • 13 • 14 • 15 | ||
All AIME Problems and Solutions |
The problems on this page are copyrighted by the Mathematical Association of America's American Mathematics Competitions.
IMO 1999